Request for help to find a closed-form expression for...












11












$begingroup$


By the comparison test, the series converges.
$$sum_{n=0}^{infty}frac{(2n+15)^2}{(2n+11)^3sqrt{(2n+13)}}=l$$
$$lsimeq0.39483198670640570922670209458869656945532664947383304288146572043505953641$$
(74 digits displayed)



I would like to know if it is possible to find a closed-form expression for $l$.



Edit



I used a PARI/GP function (sumnum) to compute an approximate value of $l$:



 gp > #
timer = 1 (on)
gp > p 74
realprecision = 77 significant digits (74 digits displayed)
gp > sumnum(n=0, (2*n+15)^2/(2*n+11)^3/sqrt(2*n+13),sumnuminit([+oo,-1.5]))
time = 15 ms.
%1 = 0.39483198670640570922670209458869656945532664947383304288146572043505953641
gp > sum(k=0,97,binomial(-1/2,k)*(zetahurwitz(k+3/2,11/2)+4*zetahurwitz(k+5/2,11/2)+4*zetahurwitz(k+7/2,11/2)))/2/sqrt(2)
time = 62 ms.
%2 = 0.39483198670640570922670209458869656945532664947383304288146572043505953641
gp > p 400
realprecision = 404 significant digits (400 digits displayed)
gp > sumnum(n=0, (2*n+15)^2/(2*n+11)^3/sqrt(2*n+13),sumnuminit([+oo,-1.5]))
time = 1,078 ms.
%3 = 0.3948319867064057092267020945886965694553266494738330428814657204350595364122729251535099721677148100865638904442238183504239633022203492259575689525638233484603909542517059855919506733374353926795281971622814159109637670427399334579124116308394274900167149611375839262362929064148759669151317574581119034347857659144521214162698550204609725028401793070583608454779328237780415473232520384318559950198
gp > sum(k=0,537,binomial(-1/2,k)*(zetahurwitz(k+3/2,11/2)+4*zetahurwitz(k+5/2,11/2)+4*zetahurwitz(k+7/2,11/2)))/2/sqrt(2)
time = 3,532 ms.
%4 = 0.3948319867064057092267020945886965694553266494738330428814657204350595364122729251535099721677148100865638904442238183504239633022203492259575689525638233484603909542517059855919506733374353926795281971622814159109637670427399334579124116308394274900167149611375839262362929064148759669151317574581119034347857659144521214162698550204609725028401793070583608454779328237780415473232520384318559950198









share|cite|improve this question











$endgroup$








  • 6




    $begingroup$
    Do you have any reason for assuming that it does indeed have a closed form?
    $endgroup$
    – Cyclohexanol.
    Dec 19 '18 at 3:57












  • $begingroup$
    whomever can answer this deserves a bounty
    $endgroup$
    – clathratus
    Dec 19 '18 at 4:33






  • 1




    $begingroup$
    Just out of curiosity : how did you compute $l$ (what tool) and how many terms did you sum (I suppose billions) ? Cheers.
    $endgroup$
    – Claude Leibovici
    Dec 20 '18 at 5:11










  • $begingroup$
    To Claude Leibovici. Even if previous comments suggest that it may be impossible to find a closed-form expression for l, your contributions are very interesting. Kudos to you.
    $endgroup$
    – jsafra
    Dec 20 '18 at 22:33










  • $begingroup$
    Did you use zetahurwitz(.) in your initial calculation to get $l$ ? If you used the original summation, how many terms did you use for $74$ decimal places ?
    $endgroup$
    – Claude Leibovici
    Dec 21 '18 at 4:18


















11












$begingroup$


By the comparison test, the series converges.
$$sum_{n=0}^{infty}frac{(2n+15)^2}{(2n+11)^3sqrt{(2n+13)}}=l$$
$$lsimeq0.39483198670640570922670209458869656945532664947383304288146572043505953641$$
(74 digits displayed)



I would like to know if it is possible to find a closed-form expression for $l$.



Edit



I used a PARI/GP function (sumnum) to compute an approximate value of $l$:



 gp > #
timer = 1 (on)
gp > p 74
realprecision = 77 significant digits (74 digits displayed)
gp > sumnum(n=0, (2*n+15)^2/(2*n+11)^3/sqrt(2*n+13),sumnuminit([+oo,-1.5]))
time = 15 ms.
%1 = 0.39483198670640570922670209458869656945532664947383304288146572043505953641
gp > sum(k=0,97,binomial(-1/2,k)*(zetahurwitz(k+3/2,11/2)+4*zetahurwitz(k+5/2,11/2)+4*zetahurwitz(k+7/2,11/2)))/2/sqrt(2)
time = 62 ms.
%2 = 0.39483198670640570922670209458869656945532664947383304288146572043505953641
gp > p 400
realprecision = 404 significant digits (400 digits displayed)
gp > sumnum(n=0, (2*n+15)^2/(2*n+11)^3/sqrt(2*n+13),sumnuminit([+oo,-1.5]))
time = 1,078 ms.
%3 = 0.3948319867064057092267020945886965694553266494738330428814657204350595364122729251535099721677148100865638904442238183504239633022203492259575689525638233484603909542517059855919506733374353926795281971622814159109637670427399334579124116308394274900167149611375839262362929064148759669151317574581119034347857659144521214162698550204609725028401793070583608454779328237780415473232520384318559950198
gp > sum(k=0,537,binomial(-1/2,k)*(zetahurwitz(k+3/2,11/2)+4*zetahurwitz(k+5/2,11/2)+4*zetahurwitz(k+7/2,11/2)))/2/sqrt(2)
time = 3,532 ms.
%4 = 0.3948319867064057092267020945886965694553266494738330428814657204350595364122729251535099721677148100865638904442238183504239633022203492259575689525638233484603909542517059855919506733374353926795281971622814159109637670427399334579124116308394274900167149611375839262362929064148759669151317574581119034347857659144521214162698550204609725028401793070583608454779328237780415473232520384318559950198









share|cite|improve this question











$endgroup$








  • 6




    $begingroup$
    Do you have any reason for assuming that it does indeed have a closed form?
    $endgroup$
    – Cyclohexanol.
    Dec 19 '18 at 3:57












  • $begingroup$
    whomever can answer this deserves a bounty
    $endgroup$
    – clathratus
    Dec 19 '18 at 4:33






  • 1




    $begingroup$
    Just out of curiosity : how did you compute $l$ (what tool) and how many terms did you sum (I suppose billions) ? Cheers.
    $endgroup$
    – Claude Leibovici
    Dec 20 '18 at 5:11










  • $begingroup$
    To Claude Leibovici. Even if previous comments suggest that it may be impossible to find a closed-form expression for l, your contributions are very interesting. Kudos to you.
    $endgroup$
    – jsafra
    Dec 20 '18 at 22:33










  • $begingroup$
    Did you use zetahurwitz(.) in your initial calculation to get $l$ ? If you used the original summation, how many terms did you use for $74$ decimal places ?
    $endgroup$
    – Claude Leibovici
    Dec 21 '18 at 4:18
















11












11








11


6



$begingroup$


By the comparison test, the series converges.
$$sum_{n=0}^{infty}frac{(2n+15)^2}{(2n+11)^3sqrt{(2n+13)}}=l$$
$$lsimeq0.39483198670640570922670209458869656945532664947383304288146572043505953641$$
(74 digits displayed)



I would like to know if it is possible to find a closed-form expression for $l$.



Edit



I used a PARI/GP function (sumnum) to compute an approximate value of $l$:



 gp > #
timer = 1 (on)
gp > p 74
realprecision = 77 significant digits (74 digits displayed)
gp > sumnum(n=0, (2*n+15)^2/(2*n+11)^3/sqrt(2*n+13),sumnuminit([+oo,-1.5]))
time = 15 ms.
%1 = 0.39483198670640570922670209458869656945532664947383304288146572043505953641
gp > sum(k=0,97,binomial(-1/2,k)*(zetahurwitz(k+3/2,11/2)+4*zetahurwitz(k+5/2,11/2)+4*zetahurwitz(k+7/2,11/2)))/2/sqrt(2)
time = 62 ms.
%2 = 0.39483198670640570922670209458869656945532664947383304288146572043505953641
gp > p 400
realprecision = 404 significant digits (400 digits displayed)
gp > sumnum(n=0, (2*n+15)^2/(2*n+11)^3/sqrt(2*n+13),sumnuminit([+oo,-1.5]))
time = 1,078 ms.
%3 = 0.3948319867064057092267020945886965694553266494738330428814657204350595364122729251535099721677148100865638904442238183504239633022203492259575689525638233484603909542517059855919506733374353926795281971622814159109637670427399334579124116308394274900167149611375839262362929064148759669151317574581119034347857659144521214162698550204609725028401793070583608454779328237780415473232520384318559950198
gp > sum(k=0,537,binomial(-1/2,k)*(zetahurwitz(k+3/2,11/2)+4*zetahurwitz(k+5/2,11/2)+4*zetahurwitz(k+7/2,11/2)))/2/sqrt(2)
time = 3,532 ms.
%4 = 0.3948319867064057092267020945886965694553266494738330428814657204350595364122729251535099721677148100865638904442238183504239633022203492259575689525638233484603909542517059855919506733374353926795281971622814159109637670427399334579124116308394274900167149611375839262362929064148759669151317574581119034347857659144521214162698550204609725028401793070583608454779328237780415473232520384318559950198









share|cite|improve this question











$endgroup$




By the comparison test, the series converges.
$$sum_{n=0}^{infty}frac{(2n+15)^2}{(2n+11)^3sqrt{(2n+13)}}=l$$
$$lsimeq0.39483198670640570922670209458869656945532664947383304288146572043505953641$$
(74 digits displayed)



I would like to know if it is possible to find a closed-form expression for $l$.



Edit



I used a PARI/GP function (sumnum) to compute an approximate value of $l$:



 gp > #
timer = 1 (on)
gp > p 74
realprecision = 77 significant digits (74 digits displayed)
gp > sumnum(n=0, (2*n+15)^2/(2*n+11)^3/sqrt(2*n+13),sumnuminit([+oo,-1.5]))
time = 15 ms.
%1 = 0.39483198670640570922670209458869656945532664947383304288146572043505953641
gp > sum(k=0,97,binomial(-1/2,k)*(zetahurwitz(k+3/2,11/2)+4*zetahurwitz(k+5/2,11/2)+4*zetahurwitz(k+7/2,11/2)))/2/sqrt(2)
time = 62 ms.
%2 = 0.39483198670640570922670209458869656945532664947383304288146572043505953641
gp > p 400
realprecision = 404 significant digits (400 digits displayed)
gp > sumnum(n=0, (2*n+15)^2/(2*n+11)^3/sqrt(2*n+13),sumnuminit([+oo,-1.5]))
time = 1,078 ms.
%3 = 0.3948319867064057092267020945886965694553266494738330428814657204350595364122729251535099721677148100865638904442238183504239633022203492259575689525638233484603909542517059855919506733374353926795281971622814159109637670427399334579124116308394274900167149611375839262362929064148759669151317574581119034347857659144521214162698550204609725028401793070583608454779328237780415473232520384318559950198
gp > sum(k=0,537,binomial(-1/2,k)*(zetahurwitz(k+3/2,11/2)+4*zetahurwitz(k+5/2,11/2)+4*zetahurwitz(k+7/2,11/2)))/2/sqrt(2)
time = 3,532 ms.
%4 = 0.3948319867064057092267020945886965694553266494738330428814657204350595364122729251535099721677148100865638904442238183504239633022203492259575689525638233484603909542517059855919506733374353926795281971622814159109637670427399334579124116308394274900167149611375839262362929064148759669151317574581119034347857659144521214162698550204609725028401793070583608454779328237780415473232520384318559950198






sequences-and-series closed-form






share|cite|improve this question















share|cite|improve this question













share|cite|improve this question




share|cite|improve this question








edited Dec 20 '18 at 22:25







jsafra

















asked Dec 19 '18 at 3:10









jsafrajsafra

766




766








  • 6




    $begingroup$
    Do you have any reason for assuming that it does indeed have a closed form?
    $endgroup$
    – Cyclohexanol.
    Dec 19 '18 at 3:57












  • $begingroup$
    whomever can answer this deserves a bounty
    $endgroup$
    – clathratus
    Dec 19 '18 at 4:33






  • 1




    $begingroup$
    Just out of curiosity : how did you compute $l$ (what tool) and how many terms did you sum (I suppose billions) ? Cheers.
    $endgroup$
    – Claude Leibovici
    Dec 20 '18 at 5:11










  • $begingroup$
    To Claude Leibovici. Even if previous comments suggest that it may be impossible to find a closed-form expression for l, your contributions are very interesting. Kudos to you.
    $endgroup$
    – jsafra
    Dec 20 '18 at 22:33










  • $begingroup$
    Did you use zetahurwitz(.) in your initial calculation to get $l$ ? If you used the original summation, how many terms did you use for $74$ decimal places ?
    $endgroup$
    – Claude Leibovici
    Dec 21 '18 at 4:18
















  • 6




    $begingroup$
    Do you have any reason for assuming that it does indeed have a closed form?
    $endgroup$
    – Cyclohexanol.
    Dec 19 '18 at 3:57












  • $begingroup$
    whomever can answer this deserves a bounty
    $endgroup$
    – clathratus
    Dec 19 '18 at 4:33






  • 1




    $begingroup$
    Just out of curiosity : how did you compute $l$ (what tool) and how many terms did you sum (I suppose billions) ? Cheers.
    $endgroup$
    – Claude Leibovici
    Dec 20 '18 at 5:11










  • $begingroup$
    To Claude Leibovici. Even if previous comments suggest that it may be impossible to find a closed-form expression for l, your contributions are very interesting. Kudos to you.
    $endgroup$
    – jsafra
    Dec 20 '18 at 22:33










  • $begingroup$
    Did you use zetahurwitz(.) in your initial calculation to get $l$ ? If you used the original summation, how many terms did you use for $74$ decimal places ?
    $endgroup$
    – Claude Leibovici
    Dec 21 '18 at 4:18










6




6




$begingroup$
Do you have any reason for assuming that it does indeed have a closed form?
$endgroup$
– Cyclohexanol.
Dec 19 '18 at 3:57






$begingroup$
Do you have any reason for assuming that it does indeed have a closed form?
$endgroup$
– Cyclohexanol.
Dec 19 '18 at 3:57














$begingroup$
whomever can answer this deserves a bounty
$endgroup$
– clathratus
Dec 19 '18 at 4:33




$begingroup$
whomever can answer this deserves a bounty
$endgroup$
– clathratus
Dec 19 '18 at 4:33




1




1




$begingroup$
Just out of curiosity : how did you compute $l$ (what tool) and how many terms did you sum (I suppose billions) ? Cheers.
$endgroup$
– Claude Leibovici
Dec 20 '18 at 5:11




$begingroup$
Just out of curiosity : how did you compute $l$ (what tool) and how many terms did you sum (I suppose billions) ? Cheers.
$endgroup$
– Claude Leibovici
Dec 20 '18 at 5:11












$begingroup$
To Claude Leibovici. Even if previous comments suggest that it may be impossible to find a closed-form expression for l, your contributions are very interesting. Kudos to you.
$endgroup$
– jsafra
Dec 20 '18 at 22:33




$begingroup$
To Claude Leibovici. Even if previous comments suggest that it may be impossible to find a closed-form expression for l, your contributions are very interesting. Kudos to you.
$endgroup$
– jsafra
Dec 20 '18 at 22:33












$begingroup$
Did you use zetahurwitz(.) in your initial calculation to get $l$ ? If you used the original summation, how many terms did you use for $74$ decimal places ?
$endgroup$
– Claude Leibovici
Dec 21 '18 at 4:18






$begingroup$
Did you use zetahurwitz(.) in your initial calculation to get $l$ ? If you used the original summation, how many terms did you use for $74$ decimal places ?
$endgroup$
– Claude Leibovici
Dec 21 '18 at 4:18












1 Answer
1






active

oldest

votes


















10












$begingroup$

Not a closed form but a rather good approximation of it.



Rewriting as $$a_n=frac{(2n+15)^2}{(2n+11)^3sqrt{2n+13}}=frac{(2n+15)^2}{(2n+11)^3sqrt{2n+11+2}}=frac{(2n+15)^2}{(2n+11)^{frac 72}sqrt{1+frac 2{2n+11}}}$$ we can write
$$frac 1{sqrt{1+frac 2{2n+11}}}=sum_{k=0}^infty binom{-frac{1}{2}}{k}frac{2^k}{(2n+11)^k}$$ and then face summations of terms
$$S_k=sum_{n=0}^inftyfrac{(2n+15)^2}{(2n+11)^{k+frac 72}}=2^{-k-frac{3}{2}} left(zeta left(k+frac{3}{2},frac{11}{2}right)+4 zeta
left(k+frac{5}{2},frac{11}{2}right)+4 zeta
left(k+frac{7}{2},frac{11}{2}right)right)$$
and then for the considered summation
$$Sigma=sum_{n=0}^infty frac{(2n+15)^2}{(2n+11)^3sqrt{2n+13}}$$ So, the partial sums
$$Sigma_p=frac 1{2sqrt 2}sum_{k=0}^p binom{-frac{1}{2}}{k} left(zeta left(k+frac{3}{2},frac{11}{2}right)+4 zeta
left(k+frac{5}{2},frac{11}{2}right)+4 zeta
left(k+frac{7}{2},frac{11}{2}right)right)$$
Computing
$$left(
begin{array}{cc}
p & Sigma_p \
5 & 0.39483148307398830445048206504782536496698034745960346801382376024 \
10 & 0.39483198676616324735777911451436673248113391244073226133701558847 \
15 & 0.39483198670639658527502248284985700122310258168712958250026452734 \
20 & 0.39483198670640571076206255275333399438974388443990146144695977886 \
25 & 0.39483198670640570922643131344152894240169346679460070946980052257 \
30 & 0.39483198670640570922670214360763624644791582367286943018650562365 \
35 & 0.39483198670640570922670209457967764179263574295643280410747243813 \
40 & 0.39483198670640570922670209458869824722904384200792304484978282169 \
45 & 0.39483198670640570922670209458869656914069678422236918330533995116 \
50 & 0.39483198670640570922670209458869656945538601634396101492730552390 \
55 & 0.39483198670640570922670209458869656945532663821679612114844784572 \
60 & 0.39483198670640570922670209458869656945532664947597620995355367822 \
65 & 0.39483198670640570922670209458869656945532664947383263347402134364 \
70 & 0.39483198670640570922670209458869656945532664947383304295989988132 \
75 & 0.39483198670640570922670209458869656945532664947383304288145065669 \
80 & 0.39483198670640570922670209458869656945532664947383304288146572333 \
85 & 0.39483198670640570922670209458869656945532664947383304288146572043 \
90 & 0.39483198670640570922670209458869656945532664947383304288146572044 \
95 & 0.39483198670640570922670209458869656945532664947383304288146572044 \
100 & 0.39483198670640570922670209458869656945532664947383304288146572044
end{array}
right)$$



Edit



In order to know how many terms have to be added for $p$ significant digits, since we face an alternatin series, consider
$$a_k=frac 1{2sqrt 2}binom{-frac{1}{2}}{k} left(zeta left(k+frac{3}{2},frac{11}{2}right)+4 zeta
left(k+frac{5}{2},frac{11}{2}right)+4 zeta
left(k+frac{7}{2},frac{11}{2}right)right)$$
A quick and dirty linear regression (built for $10 leq k leq 1000$ by steps of $10$) shows that (and this is an almost perfect fit)
$$log_{10}(|a_k|)=-0.740989, k-2.51445$$ So, for $p$ exact digits, we need to sum up $lceil 1.35, p -3.40 rceil$ terms (just as reflected by the values in the above table). For $74$ digits as given in the post, then $k=97$ which has been verified. Using the original summation, I suppose that billions of terms have been added (summing from $n=0$ to $n=10^9$ leading to $0.39481$). This look normal since, for large values of $n$, $frac{a_{n+1}}{a_n}=1-frac{3}{2 n}+Oleft(frac{1}{n^2}right)$ which is extremely slow.






share|cite|improve this answer











$endgroup$













  • $begingroup$
    What field of math is this? it's fascinating
    $endgroup$
    – clathratus
    Dec 20 '18 at 22:37










  • $begingroup$
    Claude Leibovici's work is often, as in this case, quite impressive.
    $endgroup$
    – marty cohen
    Dec 20 '18 at 23:06










  • $begingroup$
    @martycohen. Be sure that, this coming from you, I really appreciate ! Thanks.
    $endgroup$
    – Claude Leibovici
    Dec 21 '18 at 4:12










  • $begingroup$
    @clathratus. At least, part of the ones I do enjoy ! Cheers.
    $endgroup$
    – Claude Leibovici
    Dec 21 '18 at 4:29












Your Answer





StackExchange.ifUsing("editor", function () {
return StackExchange.using("mathjaxEditing", function () {
StackExchange.MarkdownEditor.creationCallbacks.add(function (editor, postfix) {
StackExchange.mathjaxEditing.prepareWmdForMathJax(editor, postfix, [["$", "$"], ["\\(","\\)"]]);
});
});
}, "mathjax-editing");

StackExchange.ready(function() {
var channelOptions = {
tags: "".split(" "),
id: "69"
};
initTagRenderer("".split(" "), "".split(" "), channelOptions);

StackExchange.using("externalEditor", function() {
// Have to fire editor after snippets, if snippets enabled
if (StackExchange.settings.snippets.snippetsEnabled) {
StackExchange.using("snippets", function() {
createEditor();
});
}
else {
createEditor();
}
});

function createEditor() {
StackExchange.prepareEditor({
heartbeatType: 'answer',
autoActivateHeartbeat: false,
convertImagesToLinks: true,
noModals: true,
showLowRepImageUploadWarning: true,
reputationToPostImages: 10,
bindNavPrevention: true,
postfix: "",
imageUploader: {
brandingHtml: "Powered by u003ca class="icon-imgur-white" href="https://imgur.com/"u003eu003c/au003e",
contentPolicyHtml: "User contributions licensed under u003ca href="https://creativecommons.org/licenses/by-sa/3.0/"u003ecc by-sa 3.0 with attribution requiredu003c/au003e u003ca href="https://stackoverflow.com/legal/content-policy"u003e(content policy)u003c/au003e",
allowUrls: true
},
noCode: true, onDemand: true,
discardSelector: ".discard-answer"
,immediatelyShowMarkdownHelp:true
});


}
});














draft saved

draft discarded


















StackExchange.ready(
function () {
StackExchange.openid.initPostLogin('.new-post-login', 'https%3a%2f%2fmath.stackexchange.com%2fquestions%2f3045951%2frequest-for-help-to-find-a-closed-form-expression-for-sum-n-0-infty-frac%23new-answer', 'question_page');
}
);

Post as a guest















Required, but never shown

























1 Answer
1






active

oldest

votes








1 Answer
1






active

oldest

votes









active

oldest

votes






active

oldest

votes









10












$begingroup$

Not a closed form but a rather good approximation of it.



Rewriting as $$a_n=frac{(2n+15)^2}{(2n+11)^3sqrt{2n+13}}=frac{(2n+15)^2}{(2n+11)^3sqrt{2n+11+2}}=frac{(2n+15)^2}{(2n+11)^{frac 72}sqrt{1+frac 2{2n+11}}}$$ we can write
$$frac 1{sqrt{1+frac 2{2n+11}}}=sum_{k=0}^infty binom{-frac{1}{2}}{k}frac{2^k}{(2n+11)^k}$$ and then face summations of terms
$$S_k=sum_{n=0}^inftyfrac{(2n+15)^2}{(2n+11)^{k+frac 72}}=2^{-k-frac{3}{2}} left(zeta left(k+frac{3}{2},frac{11}{2}right)+4 zeta
left(k+frac{5}{2},frac{11}{2}right)+4 zeta
left(k+frac{7}{2},frac{11}{2}right)right)$$
and then for the considered summation
$$Sigma=sum_{n=0}^infty frac{(2n+15)^2}{(2n+11)^3sqrt{2n+13}}$$ So, the partial sums
$$Sigma_p=frac 1{2sqrt 2}sum_{k=0}^p binom{-frac{1}{2}}{k} left(zeta left(k+frac{3}{2},frac{11}{2}right)+4 zeta
left(k+frac{5}{2},frac{11}{2}right)+4 zeta
left(k+frac{7}{2},frac{11}{2}right)right)$$
Computing
$$left(
begin{array}{cc}
p & Sigma_p \
5 & 0.39483148307398830445048206504782536496698034745960346801382376024 \
10 & 0.39483198676616324735777911451436673248113391244073226133701558847 \
15 & 0.39483198670639658527502248284985700122310258168712958250026452734 \
20 & 0.39483198670640571076206255275333399438974388443990146144695977886 \
25 & 0.39483198670640570922643131344152894240169346679460070946980052257 \
30 & 0.39483198670640570922670214360763624644791582367286943018650562365 \
35 & 0.39483198670640570922670209457967764179263574295643280410747243813 \
40 & 0.39483198670640570922670209458869824722904384200792304484978282169 \
45 & 0.39483198670640570922670209458869656914069678422236918330533995116 \
50 & 0.39483198670640570922670209458869656945538601634396101492730552390 \
55 & 0.39483198670640570922670209458869656945532663821679612114844784572 \
60 & 0.39483198670640570922670209458869656945532664947597620995355367822 \
65 & 0.39483198670640570922670209458869656945532664947383263347402134364 \
70 & 0.39483198670640570922670209458869656945532664947383304295989988132 \
75 & 0.39483198670640570922670209458869656945532664947383304288145065669 \
80 & 0.39483198670640570922670209458869656945532664947383304288146572333 \
85 & 0.39483198670640570922670209458869656945532664947383304288146572043 \
90 & 0.39483198670640570922670209458869656945532664947383304288146572044 \
95 & 0.39483198670640570922670209458869656945532664947383304288146572044 \
100 & 0.39483198670640570922670209458869656945532664947383304288146572044
end{array}
right)$$



Edit



In order to know how many terms have to be added for $p$ significant digits, since we face an alternatin series, consider
$$a_k=frac 1{2sqrt 2}binom{-frac{1}{2}}{k} left(zeta left(k+frac{3}{2},frac{11}{2}right)+4 zeta
left(k+frac{5}{2},frac{11}{2}right)+4 zeta
left(k+frac{7}{2},frac{11}{2}right)right)$$
A quick and dirty linear regression (built for $10 leq k leq 1000$ by steps of $10$) shows that (and this is an almost perfect fit)
$$log_{10}(|a_k|)=-0.740989, k-2.51445$$ So, for $p$ exact digits, we need to sum up $lceil 1.35, p -3.40 rceil$ terms (just as reflected by the values in the above table). For $74$ digits as given in the post, then $k=97$ which has been verified. Using the original summation, I suppose that billions of terms have been added (summing from $n=0$ to $n=10^9$ leading to $0.39481$). This look normal since, for large values of $n$, $frac{a_{n+1}}{a_n}=1-frac{3}{2 n}+Oleft(frac{1}{n^2}right)$ which is extremely slow.






share|cite|improve this answer











$endgroup$













  • $begingroup$
    What field of math is this? it's fascinating
    $endgroup$
    – clathratus
    Dec 20 '18 at 22:37










  • $begingroup$
    Claude Leibovici's work is often, as in this case, quite impressive.
    $endgroup$
    – marty cohen
    Dec 20 '18 at 23:06










  • $begingroup$
    @martycohen. Be sure that, this coming from you, I really appreciate ! Thanks.
    $endgroup$
    – Claude Leibovici
    Dec 21 '18 at 4:12










  • $begingroup$
    @clathratus. At least, part of the ones I do enjoy ! Cheers.
    $endgroup$
    – Claude Leibovici
    Dec 21 '18 at 4:29
















10












$begingroup$

Not a closed form but a rather good approximation of it.



Rewriting as $$a_n=frac{(2n+15)^2}{(2n+11)^3sqrt{2n+13}}=frac{(2n+15)^2}{(2n+11)^3sqrt{2n+11+2}}=frac{(2n+15)^2}{(2n+11)^{frac 72}sqrt{1+frac 2{2n+11}}}$$ we can write
$$frac 1{sqrt{1+frac 2{2n+11}}}=sum_{k=0}^infty binom{-frac{1}{2}}{k}frac{2^k}{(2n+11)^k}$$ and then face summations of terms
$$S_k=sum_{n=0}^inftyfrac{(2n+15)^2}{(2n+11)^{k+frac 72}}=2^{-k-frac{3}{2}} left(zeta left(k+frac{3}{2},frac{11}{2}right)+4 zeta
left(k+frac{5}{2},frac{11}{2}right)+4 zeta
left(k+frac{7}{2},frac{11}{2}right)right)$$
and then for the considered summation
$$Sigma=sum_{n=0}^infty frac{(2n+15)^2}{(2n+11)^3sqrt{2n+13}}$$ So, the partial sums
$$Sigma_p=frac 1{2sqrt 2}sum_{k=0}^p binom{-frac{1}{2}}{k} left(zeta left(k+frac{3}{2},frac{11}{2}right)+4 zeta
left(k+frac{5}{2},frac{11}{2}right)+4 zeta
left(k+frac{7}{2},frac{11}{2}right)right)$$
Computing
$$left(
begin{array}{cc}
p & Sigma_p \
5 & 0.39483148307398830445048206504782536496698034745960346801382376024 \
10 & 0.39483198676616324735777911451436673248113391244073226133701558847 \
15 & 0.39483198670639658527502248284985700122310258168712958250026452734 \
20 & 0.39483198670640571076206255275333399438974388443990146144695977886 \
25 & 0.39483198670640570922643131344152894240169346679460070946980052257 \
30 & 0.39483198670640570922670214360763624644791582367286943018650562365 \
35 & 0.39483198670640570922670209457967764179263574295643280410747243813 \
40 & 0.39483198670640570922670209458869824722904384200792304484978282169 \
45 & 0.39483198670640570922670209458869656914069678422236918330533995116 \
50 & 0.39483198670640570922670209458869656945538601634396101492730552390 \
55 & 0.39483198670640570922670209458869656945532663821679612114844784572 \
60 & 0.39483198670640570922670209458869656945532664947597620995355367822 \
65 & 0.39483198670640570922670209458869656945532664947383263347402134364 \
70 & 0.39483198670640570922670209458869656945532664947383304295989988132 \
75 & 0.39483198670640570922670209458869656945532664947383304288145065669 \
80 & 0.39483198670640570922670209458869656945532664947383304288146572333 \
85 & 0.39483198670640570922670209458869656945532664947383304288146572043 \
90 & 0.39483198670640570922670209458869656945532664947383304288146572044 \
95 & 0.39483198670640570922670209458869656945532664947383304288146572044 \
100 & 0.39483198670640570922670209458869656945532664947383304288146572044
end{array}
right)$$



Edit



In order to know how many terms have to be added for $p$ significant digits, since we face an alternatin series, consider
$$a_k=frac 1{2sqrt 2}binom{-frac{1}{2}}{k} left(zeta left(k+frac{3}{2},frac{11}{2}right)+4 zeta
left(k+frac{5}{2},frac{11}{2}right)+4 zeta
left(k+frac{7}{2},frac{11}{2}right)right)$$
A quick and dirty linear regression (built for $10 leq k leq 1000$ by steps of $10$) shows that (and this is an almost perfect fit)
$$log_{10}(|a_k|)=-0.740989, k-2.51445$$ So, for $p$ exact digits, we need to sum up $lceil 1.35, p -3.40 rceil$ terms (just as reflected by the values in the above table). For $74$ digits as given in the post, then $k=97$ which has been verified. Using the original summation, I suppose that billions of terms have been added (summing from $n=0$ to $n=10^9$ leading to $0.39481$). This look normal since, for large values of $n$, $frac{a_{n+1}}{a_n}=1-frac{3}{2 n}+Oleft(frac{1}{n^2}right)$ which is extremely slow.






share|cite|improve this answer











$endgroup$













  • $begingroup$
    What field of math is this? it's fascinating
    $endgroup$
    – clathratus
    Dec 20 '18 at 22:37










  • $begingroup$
    Claude Leibovici's work is often, as in this case, quite impressive.
    $endgroup$
    – marty cohen
    Dec 20 '18 at 23:06










  • $begingroup$
    @martycohen. Be sure that, this coming from you, I really appreciate ! Thanks.
    $endgroup$
    – Claude Leibovici
    Dec 21 '18 at 4:12










  • $begingroup$
    @clathratus. At least, part of the ones I do enjoy ! Cheers.
    $endgroup$
    – Claude Leibovici
    Dec 21 '18 at 4:29














10












10








10





$begingroup$

Not a closed form but a rather good approximation of it.



Rewriting as $$a_n=frac{(2n+15)^2}{(2n+11)^3sqrt{2n+13}}=frac{(2n+15)^2}{(2n+11)^3sqrt{2n+11+2}}=frac{(2n+15)^2}{(2n+11)^{frac 72}sqrt{1+frac 2{2n+11}}}$$ we can write
$$frac 1{sqrt{1+frac 2{2n+11}}}=sum_{k=0}^infty binom{-frac{1}{2}}{k}frac{2^k}{(2n+11)^k}$$ and then face summations of terms
$$S_k=sum_{n=0}^inftyfrac{(2n+15)^2}{(2n+11)^{k+frac 72}}=2^{-k-frac{3}{2}} left(zeta left(k+frac{3}{2},frac{11}{2}right)+4 zeta
left(k+frac{5}{2},frac{11}{2}right)+4 zeta
left(k+frac{7}{2},frac{11}{2}right)right)$$
and then for the considered summation
$$Sigma=sum_{n=0}^infty frac{(2n+15)^2}{(2n+11)^3sqrt{2n+13}}$$ So, the partial sums
$$Sigma_p=frac 1{2sqrt 2}sum_{k=0}^p binom{-frac{1}{2}}{k} left(zeta left(k+frac{3}{2},frac{11}{2}right)+4 zeta
left(k+frac{5}{2},frac{11}{2}right)+4 zeta
left(k+frac{7}{2},frac{11}{2}right)right)$$
Computing
$$left(
begin{array}{cc}
p & Sigma_p \
5 & 0.39483148307398830445048206504782536496698034745960346801382376024 \
10 & 0.39483198676616324735777911451436673248113391244073226133701558847 \
15 & 0.39483198670639658527502248284985700122310258168712958250026452734 \
20 & 0.39483198670640571076206255275333399438974388443990146144695977886 \
25 & 0.39483198670640570922643131344152894240169346679460070946980052257 \
30 & 0.39483198670640570922670214360763624644791582367286943018650562365 \
35 & 0.39483198670640570922670209457967764179263574295643280410747243813 \
40 & 0.39483198670640570922670209458869824722904384200792304484978282169 \
45 & 0.39483198670640570922670209458869656914069678422236918330533995116 \
50 & 0.39483198670640570922670209458869656945538601634396101492730552390 \
55 & 0.39483198670640570922670209458869656945532663821679612114844784572 \
60 & 0.39483198670640570922670209458869656945532664947597620995355367822 \
65 & 0.39483198670640570922670209458869656945532664947383263347402134364 \
70 & 0.39483198670640570922670209458869656945532664947383304295989988132 \
75 & 0.39483198670640570922670209458869656945532664947383304288145065669 \
80 & 0.39483198670640570922670209458869656945532664947383304288146572333 \
85 & 0.39483198670640570922670209458869656945532664947383304288146572043 \
90 & 0.39483198670640570922670209458869656945532664947383304288146572044 \
95 & 0.39483198670640570922670209458869656945532664947383304288146572044 \
100 & 0.39483198670640570922670209458869656945532664947383304288146572044
end{array}
right)$$



Edit



In order to know how many terms have to be added for $p$ significant digits, since we face an alternatin series, consider
$$a_k=frac 1{2sqrt 2}binom{-frac{1}{2}}{k} left(zeta left(k+frac{3}{2},frac{11}{2}right)+4 zeta
left(k+frac{5}{2},frac{11}{2}right)+4 zeta
left(k+frac{7}{2},frac{11}{2}right)right)$$
A quick and dirty linear regression (built for $10 leq k leq 1000$ by steps of $10$) shows that (and this is an almost perfect fit)
$$log_{10}(|a_k|)=-0.740989, k-2.51445$$ So, for $p$ exact digits, we need to sum up $lceil 1.35, p -3.40 rceil$ terms (just as reflected by the values in the above table). For $74$ digits as given in the post, then $k=97$ which has been verified. Using the original summation, I suppose that billions of terms have been added (summing from $n=0$ to $n=10^9$ leading to $0.39481$). This look normal since, for large values of $n$, $frac{a_{n+1}}{a_n}=1-frac{3}{2 n}+Oleft(frac{1}{n^2}right)$ which is extremely slow.






share|cite|improve this answer











$endgroup$



Not a closed form but a rather good approximation of it.



Rewriting as $$a_n=frac{(2n+15)^2}{(2n+11)^3sqrt{2n+13}}=frac{(2n+15)^2}{(2n+11)^3sqrt{2n+11+2}}=frac{(2n+15)^2}{(2n+11)^{frac 72}sqrt{1+frac 2{2n+11}}}$$ we can write
$$frac 1{sqrt{1+frac 2{2n+11}}}=sum_{k=0}^infty binom{-frac{1}{2}}{k}frac{2^k}{(2n+11)^k}$$ and then face summations of terms
$$S_k=sum_{n=0}^inftyfrac{(2n+15)^2}{(2n+11)^{k+frac 72}}=2^{-k-frac{3}{2}} left(zeta left(k+frac{3}{2},frac{11}{2}right)+4 zeta
left(k+frac{5}{2},frac{11}{2}right)+4 zeta
left(k+frac{7}{2},frac{11}{2}right)right)$$
and then for the considered summation
$$Sigma=sum_{n=0}^infty frac{(2n+15)^2}{(2n+11)^3sqrt{2n+13}}$$ So, the partial sums
$$Sigma_p=frac 1{2sqrt 2}sum_{k=0}^p binom{-frac{1}{2}}{k} left(zeta left(k+frac{3}{2},frac{11}{2}right)+4 zeta
left(k+frac{5}{2},frac{11}{2}right)+4 zeta
left(k+frac{7}{2},frac{11}{2}right)right)$$
Computing
$$left(
begin{array}{cc}
p & Sigma_p \
5 & 0.39483148307398830445048206504782536496698034745960346801382376024 \
10 & 0.39483198676616324735777911451436673248113391244073226133701558847 \
15 & 0.39483198670639658527502248284985700122310258168712958250026452734 \
20 & 0.39483198670640571076206255275333399438974388443990146144695977886 \
25 & 0.39483198670640570922643131344152894240169346679460070946980052257 \
30 & 0.39483198670640570922670214360763624644791582367286943018650562365 \
35 & 0.39483198670640570922670209457967764179263574295643280410747243813 \
40 & 0.39483198670640570922670209458869824722904384200792304484978282169 \
45 & 0.39483198670640570922670209458869656914069678422236918330533995116 \
50 & 0.39483198670640570922670209458869656945538601634396101492730552390 \
55 & 0.39483198670640570922670209458869656945532663821679612114844784572 \
60 & 0.39483198670640570922670209458869656945532664947597620995355367822 \
65 & 0.39483198670640570922670209458869656945532664947383263347402134364 \
70 & 0.39483198670640570922670209458869656945532664947383304295989988132 \
75 & 0.39483198670640570922670209458869656945532664947383304288145065669 \
80 & 0.39483198670640570922670209458869656945532664947383304288146572333 \
85 & 0.39483198670640570922670209458869656945532664947383304288146572043 \
90 & 0.39483198670640570922670209458869656945532664947383304288146572044 \
95 & 0.39483198670640570922670209458869656945532664947383304288146572044 \
100 & 0.39483198670640570922670209458869656945532664947383304288146572044
end{array}
right)$$



Edit



In order to know how many terms have to be added for $p$ significant digits, since we face an alternatin series, consider
$$a_k=frac 1{2sqrt 2}binom{-frac{1}{2}}{k} left(zeta left(k+frac{3}{2},frac{11}{2}right)+4 zeta
left(k+frac{5}{2},frac{11}{2}right)+4 zeta
left(k+frac{7}{2},frac{11}{2}right)right)$$
A quick and dirty linear regression (built for $10 leq k leq 1000$ by steps of $10$) shows that (and this is an almost perfect fit)
$$log_{10}(|a_k|)=-0.740989, k-2.51445$$ So, for $p$ exact digits, we need to sum up $lceil 1.35, p -3.40 rceil$ terms (just as reflected by the values in the above table). For $74$ digits as given in the post, then $k=97$ which has been verified. Using the original summation, I suppose that billions of terms have been added (summing from $n=0$ to $n=10^9$ leading to $0.39481$). This look normal since, for large values of $n$, $frac{a_{n+1}}{a_n}=1-frac{3}{2 n}+Oleft(frac{1}{n^2}right)$ which is extremely slow.







share|cite|improve this answer














share|cite|improve this answer



share|cite|improve this answer








edited Dec 20 '18 at 23:26









Barry Cipra

60.5k655129




60.5k655129










answered Dec 19 '18 at 6:00









Claude LeiboviciClaude Leibovici

125k1158136




125k1158136












  • $begingroup$
    What field of math is this? it's fascinating
    $endgroup$
    – clathratus
    Dec 20 '18 at 22:37










  • $begingroup$
    Claude Leibovici's work is often, as in this case, quite impressive.
    $endgroup$
    – marty cohen
    Dec 20 '18 at 23:06










  • $begingroup$
    @martycohen. Be sure that, this coming from you, I really appreciate ! Thanks.
    $endgroup$
    – Claude Leibovici
    Dec 21 '18 at 4:12










  • $begingroup$
    @clathratus. At least, part of the ones I do enjoy ! Cheers.
    $endgroup$
    – Claude Leibovici
    Dec 21 '18 at 4:29


















  • $begingroup$
    What field of math is this? it's fascinating
    $endgroup$
    – clathratus
    Dec 20 '18 at 22:37










  • $begingroup$
    Claude Leibovici's work is often, as in this case, quite impressive.
    $endgroup$
    – marty cohen
    Dec 20 '18 at 23:06










  • $begingroup$
    @martycohen. Be sure that, this coming from you, I really appreciate ! Thanks.
    $endgroup$
    – Claude Leibovici
    Dec 21 '18 at 4:12










  • $begingroup$
    @clathratus. At least, part of the ones I do enjoy ! Cheers.
    $endgroup$
    – Claude Leibovici
    Dec 21 '18 at 4:29
















$begingroup$
What field of math is this? it's fascinating
$endgroup$
– clathratus
Dec 20 '18 at 22:37




$begingroup$
What field of math is this? it's fascinating
$endgroup$
– clathratus
Dec 20 '18 at 22:37












$begingroup$
Claude Leibovici's work is often, as in this case, quite impressive.
$endgroup$
– marty cohen
Dec 20 '18 at 23:06




$begingroup$
Claude Leibovici's work is often, as in this case, quite impressive.
$endgroup$
– marty cohen
Dec 20 '18 at 23:06












$begingroup$
@martycohen. Be sure that, this coming from you, I really appreciate ! Thanks.
$endgroup$
– Claude Leibovici
Dec 21 '18 at 4:12




$begingroup$
@martycohen. Be sure that, this coming from you, I really appreciate ! Thanks.
$endgroup$
– Claude Leibovici
Dec 21 '18 at 4:12












$begingroup$
@clathratus. At least, part of the ones I do enjoy ! Cheers.
$endgroup$
– Claude Leibovici
Dec 21 '18 at 4:29




$begingroup$
@clathratus. At least, part of the ones I do enjoy ! Cheers.
$endgroup$
– Claude Leibovici
Dec 21 '18 at 4:29


















draft saved

draft discarded




















































Thanks for contributing an answer to Mathematics Stack Exchange!


  • Please be sure to answer the question. Provide details and share your research!

But avoid



  • Asking for help, clarification, or responding to other answers.

  • Making statements based on opinion; back them up with references or personal experience.


Use MathJax to format equations. MathJax reference.


To learn more, see our tips on writing great answers.




draft saved


draft discarded














StackExchange.ready(
function () {
StackExchange.openid.initPostLogin('.new-post-login', 'https%3a%2f%2fmath.stackexchange.com%2fquestions%2f3045951%2frequest-for-help-to-find-a-closed-form-expression-for-sum-n-0-infty-frac%23new-answer', 'question_page');
}
);

Post as a guest















Required, but never shown





















































Required, but never shown














Required, but never shown












Required, but never shown







Required, but never shown

































Required, but never shown














Required, but never shown












Required, but never shown







Required, but never shown







Popular posts from this blog

Plaza Victoria

In PowerPoint, is there a keyboard shortcut for bulleted / numbered list?

How to put 3 figures in Latex with 2 figures side by side and 1 below these side by side images but in...